sixyt students are competing in a video game challenge. if 2 students comete against each other in every round how many different combinations can be made for the first round of the challenge

Answers

Answer 1

Answer:

i believe its 30

Step-by-step explanation:

well i think you would divide sixty by 2 which is thirty.

IM SO SORRY IF IM WRONG- THIS IS JUST WHAT I THINK!


Related Questions

Simplify the expression by first transforming the radical to exponential form. Leave the answer in exact form as a radical or a power, not as a decimal approximation.

Answers

Answer:

[tex]\textsf{Radical form}: \quad \sqrt[4]{2}\\\\\textsf{Exponent form}: \quad 2^{\frac{1}{4}}[/tex]

Step-by-step explanation:

Given expression:

[tex]\sqrt{8} \div \sqrt[4]{32}[/tex]

[tex]\textsf{Apply the exponent rule} \quad \sqrt[n]{a}=a^{\frac{1}{n}}:[/tex]

[tex]\implies 8^{\frac{1}{2}} \div 32^{\frac{1}{4}}[/tex]

Rewrite 8 as 2³ and 32 as 2⁵:

[tex]\implies (2^3)^{\frac{1}{2}} \div (2^5)^{\frac{1}{4}}[/tex]

[tex]\textsf{Apply the exponent rule} \quad (a^b)^c=a^{bc}:[/tex]

[tex]\implies 2^{\frac{3}{2}} \div 2^{\frac{5}{4}}[/tex]

[tex]\textsf{Apply the exponent rule} \quad a^b \div a^c=a^{b-c}:[/tex]

[tex]\implies 2^{\frac{3}{2}-\frac{5}{4}}[/tex]

[tex]\implies 2^{\frac{6}{4}-\frac{5}{4}}[/tex]

[tex]\implies 2^{\frac{1}{4}}[/tex]

[tex]\textsf{Apply the exponent rule} \quad \sqrt[n]{a}=a^{\frac{1}{n}}:[/tex]

[tex]\implies \sqrt[4]{2}[/tex]

Which number line shows all the values of x that make the inequality - 3x +1 <7 true?A2-5-4--3-2-10123B.5in-4-3-2-1012.34С5-5-4-3-2-1012345D-4-3-2.12345

Answers

First let's solve the given inequality:-

[tex]\begin{gathered} -3x+1<7 \\ -3x<6 \\ x>-2 \end{gathered}[/tex]

So the correct option is (D).

1. m/ASN = 63°
m/GSN =

Answers

The measure of angle ∠GSN is 27°.

What do we mean by angles?An angle is a figure in plane geometry that is created by two rays or lines that have a common endpoint. The Latin word "angulus," which means "corner," is the source of the English word "angle." The common endpoint of two rays is known as the vertex, and the two rays are referred to as the sides of an angle.

So, a measure of ∠GSN:

The given angle ASG is 90° (Given)∠ASN = 63°

Then, ∠GSN will be:

∠ASN + ∠GSN = ∠ASG63 + ∠GSN = 90∠GSN = 90 - 63∠GSN = 27°

Therefore, the measure of angle ∠GSN is 27°.

Know more about angles here:

https://brainly.com/question/25716982

#SPJ3

please help with this practice question thank you

Answers

Answer:

Formula for slope of line is given as y2-y1 ÷ x2-x1 or y1-y2 ÷ x1-x2, where x and y are the coordinates of the points.

First, identify the coordinates of the two points shown on the graph.

First coordinate is (0,-1) and second coordinate is (3,1).

After that, find the slope of the line using the formula.

Slope = (1-(-1))÷(3-0)

= 2/3

What is the end behavior of the polynomial function?

Drag the choices into the boxes to correctly describe the end behavior of the function.
Put responses in the correct input to answer the question. Select a response, navigate to the desired input and insert the response. Responses can be selected and inserted using the space bar, enter key, left mouse button or touchpad. Responses can also be moved by dragging with a mouse.
f(x)=6x9−6x4−6 f(x)=−3x4−6x+4x−5

Answers

The end behavior of each function is given as follows:

f(x) = 6x^9 - 6x^4 - 4, as x -> -∞, f(x) -> -∞ and as x -> ∞, f(x) -> ∞.f(x) = -3x^4 - 6x + 4x - 5, as x -> -∞, f(x) -> -∞ and as x -> ∞, f(x) -> -∞.

End behavior of a function

The end behavior of a function is given by the limits of the function as x goes to infinity, both negative and positive infinity, giving how the function behaves to the left and to the right of the graph.

For a polynomial function, only the term with the highest exponent is considered for the calculation of the limit, which is a standard rule for limits when x goes to infinity.

The first function is given by:

f(x) = 6x^9 - 6x^4 - 4.

Then the limits that define the end behavior of the function are given as follows:

lim x -> -∞ x^9 = (-∞)^9 = -∞.lim x -> ∞ x^9 = (∞)^9 = ∞.

The second function is given by:

f(x) = -3x^4 - 6x + 4x - 5.

Then the limits that define the end behavior of the function are given as follows:

lim x -> -∞ -x^4 = -(-∞)^4 = -∞.lim x -> ∞ -x^4 = -(∞)^4 = -∞.

A similar problem, also about the end behavior of a function, is presented at https://brainly.com/question/28884735

#SPJ1

A right triangle has an area of 54 ft2 and a hypotenuse of 25 ft long. What are the lengths of its other two sides?

Answers

By theorem we have the following:

[tex]h^2=a^2+b^2[/tex]

And, we are given:

[tex]A=\frac{a\cdot b}{2}\Rightarrow2A=a\cdot b[/tex]

Then:

[tex]\Rightarrow4A^2=a^2b^2\Rightarrow4A^2=a^2(h^2-a^2)[/tex][tex]\Rightarrow a^4-h^2a^2+4A^2=0[/tex]

Now, we replace h and A:

[tex]a^4-(25)^2a^2+4(54)^2=0[/tex]

And solve for a:

[tex]a^4-625a^2+11664=0[/tex]

Then, the possible values for a are:

[tex]a=\begin{cases}a_1=-\frac{29}{2}-\frac{\sqrt[]{409}}{2} \\ a_2=\frac{29}{2}-\frac{\sqrt[]{409}}{2} \\ a_3=\frac{\sqrt[]{409}}{2}-\frac{29}{2} \\ a_4=\frac{29}{2}+\frac{\sqrt[]{409}}{2} \\ \end{cases}[/tex]

We can see that a1, and a2 are not solutions, therefore a2 and a4 are.

So, the two possible b sides are then:

[tex]b_2=\sqrt[]{25^2-(\frac{29}{2}-\frac{\sqrt[]{409}}{2})^2}\Rightarrow b_2\approx24.99[/tex]

and:

[tex]b_4=\sqrt[^{}]{25^2-(\frac{29}{2}+\frac{\sqrt[]{409}}{2})^2}\Rightarrow b_{4\approx}15.50[/tex]

So, the lengths of the two sides can be:

a = 4.38 and b = 24.99

or

a = 24.61 and b = 15.50

write a point slope equation for the line that has a slope 5and passes the point (6,22).

Answers

Solution:

The general equation of a line of slope m passing through a point A is expressed as

[tex]\begin{gathered} y-y_1=m(x-x_1) \\ \text{where} \\ (x_1,y_1)\text{ is the coordinate of the point A through which the line passes through} \end{gathered}[/tex]

Given that the line has a slope of 5, and passes through the (6, 22), we have

[tex]\begin{gathered} m=5 \\ x_1=6 \\ y_1=22 \end{gathered}[/tex]

thus,

[tex]\begin{gathered} y-y_1=m(x-x_1) \\ \Rightarrow y-22=5(x-6) \end{gathered}[/tex]

Hence, the point-slope equation for the line is expressed as

[tex]y-22=5(x-6)[/tex]

You plan to work for 40 years and then retire using a 25-year annuity. You want to arrange a retirement income of $4500 per month. You have access to an account that pays an APR of 8.4% compounded monthly.

Answers

The desired monthly yield at the retirement time will be equal to $565,714.28.

Compound Interest may be defined as the interest earned by the bank on the basis of principle and also accumulated interest which increases exponentially and not linearly with respect to time. In calculating compound interest, the amount earned at the end of first year becomes principle for the next year and so on. Compound interest can be calculated, annually, half-yearly or quarterly etc.

Time for which work is planned = 40 years, Principle = $4500 and APR = 8.4% = 0.084/12 = 0.007.

The value of n = 12 × 25 = 300

The amount can be calculated by the formula A = P/r [1 - (1 + r) ⁻ⁿ]

A = (4500/0.007) [1 - (1 + 0.007) ⁻³⁰⁰]

A = 642,857.14 [1 - 0.12]

A = 642,857.14 × 0.88

A = $565,714.28 which is required amount.

Learn more about Compound Interest at:

brainly.com/question/13463611

#SPJ9

Complete Question:

You plan to work for 40 years and then retire using a 25-year annuity. You want to arrange a retirement income of $4500 per month. You have access to an account that pays an APR of 8.4% compounded monthly. What monthly deposits are required to achieve the desired monthly yield at retirement?

jamals lawn is shaped like a square with an area of 224.9 ft2. which measurement is closest to the side length of his lawn in feet?

Answers

Given:

Area of the square shaped lawn = 224.9 ft²

A square has all four side lengths equal.

To find the side length of this lawn, use the formula for area of a square below:

[tex]\text{Area = }L^2[/tex]

Take the square root of both sides to find the sile length L:

[tex]\begin{gathered} \sqrt[]{Area\text{ }}=\sqrt[]{L^2} \\ \\ \sqrt[]{Area}\text{ = L} \\ \\ \sqrt[]{224.9}\text{ = L} \\ \\ 14.99\text{ ft = L} \end{gathered}[/tex]

Therefore, the measurement that is closest to the side length in feet is 15 ft

ANSWER:

15 ft

Which answer choice shows two hundred and two thousandths?A) 200.02B) 200.202C) 202.02D) 202.002

Answers

Given

two hundred two and two thousandths.

Answer

202.002

Option D is correct

Which of the expressions are equivalent to the one below? Check all that apply.

( 15 • 3) - 20

Answers

Answer:

I don't know your answer choices but...

Step-by-step explanation:

( 15 • 3) - 20 is equal to:

45-20

25

(5)(3)(3)-20

which variable has a set of zero pairs as a coefficients? (x or y)2x + 3y=20-2x + y=4

Answers

Answer:

The variable that has a set of zero pairs as a coefficients is;

[tex]x[/tex]

Explanation:

We want to find the variable that has a set of zero pairs as a coefficients.

Zero pair is a pair of number that sum up to give zero.

Given the system of equation;

[tex]\begin{gathered} 2x+3y=20 \\ -2x+y=4 \end{gathered}[/tex]

The pair of coefficient of x is;

[tex]\begin{gathered} 2\text{ and -2} \\ 2+-2=2-2=0 \end{gathered}[/tex]

The pair of coefficient of y is;

[tex]\begin{gathered} 3\text{ and 1} \\ 3+1=4 \end{gathered}[/tex]

So, since the coefficient of x sum up to give zero.

The variable that has a set of zero pairs as a coefficients is;

[tex]x[/tex]

4^5 x 4^3 x 4^2 x 4^3 / 4^2 x 4 x 4^2 simplify help

Answers

The simplification form of the given mathematical equation 4^5 x 4^3 x 4^2 x 4^3 / 4^2 x 4 x 4^2 is [tex]4^{8}[/tex]

In the given question, it is given

We know the simplification property of the division of numbers as,

Division in exponential form

[tex]\frac{a^{x} }{a^{y} }[/tex] = [tex]a^{x-y}[/tex] , and

Multiplication in exponential form

[tex]a^{x} . a^{y}[/tex] = [tex]a^{x+y}[/tex]

Similarly, we'll apply the same property to solve this question,

[tex]4^{5+3+2+3 + ( -2 -1 -2)}[/tex]

[tex]4^{5+3+2+3 - ( 2 + 1 + 2)}[/tex]

[tex]4^{13 - 5}[/tex]

[tex]4^{8}[/tex]

Hence, the simplification form of the given mathematical equation 4^5 x 4^3 x 4^2 x 4^3 / 4^2 x 4 x 4^2 is [tex]4^{8}[/tex]

To learn more about, simplification here

https://brainly.com/question/2804192

#SPJ1

I BEG YOU FOR HELP!!! Determine the relationship between the two triangles and whether or not they can be proven to be congruent.

Answers

Answer:

Step-by-step explanation:

The triangles are congruent because they follow the SSS triangle congruence postulate. As the SSS postulate says that all 3 sides of one triangle are congruent to another triangle's sides, these triangles shown have all three of their sides congruent to each other.

Answer: I am no sure but the only way to decide whether a pair of triangles are congruent would be to measure all of the sides and angles, and these triangles do not look the same so I would say that these tringles are not congruent.

Step-by-step explanation:

13. Write an equation of the line that passes through the points (-7, 6) and (3, -4 )in slope-
intercept form.

Answers

Answer:

Firstly we need to find the gradient of give two points as follows;

M= y

Answer:

Answer: y = -x - 1

Step-by-step explanation:

- Consider a straight line passing through (x, y) from the origin (0, 0). That line with a positive gradient of m and meets at a point (0, c) [y-intercept]

- It has a general equation as below;

[tex]{ \rm{y = mx + c}} \\ [/tex]

- So, consider the line given in our question; Let's find its slope m first;

[tex]{ \rm{slope = \frac{y _{2} - y _{1} }{x _{2} - x _{1} } }} \\ [/tex]

- From the points given in the question, (-7, 6) and (3, -4)

x_1 is -7x_2 is 3y_1 is 6y_2 is -4

[tex]{ \rm{m = \frac{ - 4 - 6}{3 - ( - 7)} }} \\ \\ { \rm{m = \frac{ - 10}{10} }} \\ \\ { \underline{ \rm{ \: m = - 1 \: }}}[/tex]

- Therefore, our equation so far is y = -x + c. Our line has a negative slope that means it slants from top to bottom, its origin is its y-intercept

- Consider point (3, -4);

[tex]{ \rm{y = - x + c}} \\ { \rm{ - 4 = - 3 + c}} \\ { \rm{c = - 1}}[/tex]

- y-intercept is -1

hence equation is y = -x - 1

[tex]{ \boxed{ \delta}}{ \underline{ \mathfrak{ \: \: beicker}}}[/tex]

What is the slope of the line passing through (3, 0) and (4, 0) ?

A) 0
B) 3/4
C) 4/3
D) Undefined

Answers

[tex]m=\frac{y_{2}-y_{1} }{x_{2} -x_{1} } \\m=\frac{0-0}{4-1} \\m=\frac{0}{1} \\m=0[/tex]

⇒0 divided by any number is 0

OPTION A IS THE ANSWER.

Answer:

A

Step-by-step explanation:

calculate the slope m using the slope formula

m = [tex]\frac{y_{2}-y_{1} }{x_{2}-x_{1} }[/tex]

with (x₁, y₁ ) = (3, 0 ) and (x₂, y₂ ) = (4, 0 )

m = [tex]\frac{0-0}{4-3}[/tex] = [tex]\frac{0}{1}[/tex] = 0

Please help me with my calculus homework, only question 3****

Answers

I would start by stating the Fundamental Theorem of Calculus which states that;

If a function f is continuous on a closed interval [a,b] and F is an antiderivative of f on the interval [a,b], then

[tex]\int ^b_af(x)dx=F(b)-F(a)\text{ }[/tex]

Let

[tex]\begin{gathered} f(x)=x^3-6x^{} \\ F(x)=\int f(x)dx=\int (x^3-6x)dx \end{gathered}[/tex]

Recall that;

[tex]\int x^n=\frac{x^{n+1}}{n+1},n\ne-1[/tex]

That implies that,

[tex]F(x)=\int (x^3-6x)dx=\int x^3dx-\int 6xdx=\frac{x^4}{4}-6(\frac{x^2}{2})=\frac{x^4}{4}-3x^2+C[/tex]

Applying the Fundamental Theorem of Calculus, where a=0, b=3

[tex]\begin{gathered} \int ^3_0(x^3-6x)dx=F(3)-F(0) \\ F(3)=\frac{3^4}{4}-3(3)^2+C=\frac{81}{4}-27+C=-\frac{27}{4}+C \\ F(0)=\frac{0^4}{4}-3(0)^2+C=C \\ \Rightarrow\int ^3_0(x^3-6x)dx=-\frac{27}{4}+C-C=-\frac{27}{4} \end{gathered}[/tex]

So the answer is -27/4

Using the distance formula, d = √(x2 - x1)2 + (y2 - y1)2, what is the distance between point (0, 5) and point (3, -1) rounded to the nearest tenth?

Answers

The distance between the points is 6.7 units

What is distance?

The distance between two points is the number of points between them

How to determine the distance?

The points are given as

(0, 5) and (3, -1)

The distance formula is given as

d = √(x2 - x1)^2 + (y2 - y1)^2

Substitute the given points in the above distance formula

So, we have

d = √(0 - 3)^2 + (5 + 1)^2

Evaluate the difference and the sum

d = √(-3)^2 + 6^2

Evaluate the exponents

d = √9 + 36

Evaluate the sum

d = √45

This gives

d = 6.7

Hence, the distance is 6.7 units

Read more about distance at

https://brainly.com/question/7243416

#SPJ1

Helppppp!!!! Please!!

Answers

n > 39/4 is value of quartic equation.

What does quartic equation mean?

A fourth-degree equation, often known as a quartic equation, is one that reduces a quartic polynomial to zero and has the formula: where a 0. A quartic function's derivative is a cubic function.

For a quadratic ax² + bx +c , the sign of its determinant, given by

 Δ = b²- 4ac

"determines" the nature of its roots. In particular, if Δ<0 , then the quadratic has two distinct non-real roots.

Now, we have

3z² - 9z = n - 3

3z² - 9z - (n - 3)  = 0

with determinant

Δ = (-9)² - 4 .3( n - 3 ) = 117 - 12n

Solve for  such that Δ < 0

Δ = 117 - 12n < 0 ⇒  12n > 117

n > 117/12

n > 39/4

Learn more about quartic equation

brainly.com/question/28593533

#SPJ13

These two equations look very similar at first. What is the difference in how you would solve them?



`\frac{x-2}{3}=5` `\frac{x}{3}-2=5`

Answers

The difference in how we would solve them is that there is a different order of steps.

We are given two equations.The two equations look similar, but there is a different order of steps in order to solve them.The first equation is :(x-2)/3 = 5Multiply both the sides by 3.x-2 = 15Add 2 on both sides.x = 17Hence, the solution of the first equation is x = 17.The second equation is :(x/3)-2 = 5Add 2 on both sides.x/3 = 7Multiply both the sides by 3.x = 21Hence, the solution of the second equation is x = 21.

To learn more about equations, visit :

https://brainly.com/question/10413253

#SPJ1

2^3= 8 is equivalent to log, C = D.cand D

Answers

we have

2^3= 8

Applying log both sides

log(2^3)=log(8)

Apply property of log

3log(2)=log(8)

therefore

C=3 and D=log(8)

Write a two-column proof.
4. Given: AB EF
AC DF
Prove: ABC ~ FED
Please help

Answers

Explanation:

The following is a proof that ∆ABC ~ ∆FED.

Statement . . . . Reason

1. AB║EF, AC║FD . . . . given

2. ∠BCA ≅ ∠EDF . . . . alternate exterior angles theorem

3. ∠ABC ≅ ∠FED . . . . alternate interior angles theorem

4. ∆ABC ~ ∆FED . . . . AA similarity postulate

mr. Morales mix for 4 4/5 pound of macaroni and cheese and brings it to the 5th grade party. the kids ate 3/4 of the total amount that mr. Morales brought. he took the rest home then gave 3/4 of a pound of the macaroni and cheese to Mr. kang the next day. how many pounds of macaroni and cheese is left over for mr. Morales to eat

Answers

Convert the Mixed number to an Improper fraction:

- Multiply the Whole number by the denominator.

- Add the product to the numerator.

- Use the same denominator.

Then:

[tex]4\frac{4}{5}=\frac{(4)(5)+4}{5}=\frac{20+4}{5}=\frac{24}{5}[/tex]

Then, the total amount of macaroni and cheese Mr. Morales brought was:

[tex]\frac{24}{5}lb[/tex]

After the kids ate macaronis and cheese, the amount he took home was:

[tex]\frac{24}{5}lb-(\frac{24}{5}lb)(\frac{3}{4})=\frac{6}{5}lb[/tex]

After he gave some macaroni and cheese to Mr. Kang the next day, the amount of macaroni and cheese (in pounds) left for mr. Morales to eat, is the following:

[tex]\frac{6}{5}lb-(\frac{6}{5}lb)(\frac{3}{4})=\frac{3}{10}lb[/tex]

The answer is:

[tex]\frac{3}{10}lb[/tex]

Sydney purchased a $50.00 gift for a baby shower. She uses a coupon that offers 20% off. How much will Sidney spend on the gift after the coupon?

Answers

From the scenario, the following are the pieces of information being given:

Price of Gift = $50

Discount Coupon used = 20% Off

Let's compute how much will Sidney spend on the gift after the coupon.

Step 1: Let's determine the equivalent amount of the discount.

[tex]\text{ Amount to be Discounted = Price of Gift x }\frac{Percentage\text{ of Discount}}{100}[/tex][tex]\text{ = \$50 x }\frac{20}{100}\text{ = \$50 x 0.20}[/tex][tex]\text{ = \$10}[/tex]

Step 2: Let's deduct the equivalent amount of 20% to the actual price of the gift.

[tex]\text{ = \$50 - \$10}[/tex][tex]\text{ = \$40}[/tex]

Therefore, Sydney will spend $40 on the gift after the coupon.

Given z1 = 5(cos 240° + isin 240°) and z2 = 15(cos 135° + isin 135°), what is the product of z1 and z2?

Answers

By multiplying z1 and z2, we get:

[tex]\begin{gathered} z1\times z2=5(cos240+isin240)15(cos135+isin135) \\ z1\times z2=75(cos240+isin240)(cos135+isin135) \end{gathered}[/tex]

Applying the distributive property:

[tex]\begin{gathered} z1\times z2=75(cos240+\imaginaryI s\imaginaryI n240)(cos135+\imaginaryI s\imaginaryI n135) \\ z1\times z2=75(cos240\times cos135+cos240\times isin135+\mathrm{i}s\mathrm{i}n240\times cos135+\imaginaryI s\imaginaryI n240\times\imaginaryI s\imaginaryI n135) \\ z\times1z\times2=75(cos240\times cos135+cos240\times\imaginaryI s\imaginaryI n135+\imaginaryI s\imaginaryI n240\times cos135-s\imaginaryI n240\times s\imaginaryI n135) \end{gathered}[/tex]

In order to simplify this, we can use the following trigonometric identities:

[tex]\begin{gathered} sin(\alpha+\beta)=sin(\alpha)cos(\beta)+cos(\alpha)sin(\beta) \\ cos(\alpha+\beta)=cos(\alpha)cos(\beta)-sin(\alpha)sin(\beta) \end{gathered}[/tex]

By taking β as 135 and α as 240, we can write:

[tex]\begin{gathered} is\imaginaryI n(240+135)=isin(375)=is\imaginaryI n(240)s\imaginaryI n(135)+icos(240)s\imaginaryI n(135) \\ cos(240+135)=cos(375)=cos(240)cos(135)-s\imaginaryI n(240)s\imaginaryI n(135) \end{gathered}[/tex]

Then, by grouping some terms of the expression, we get:

[tex]z\times1z\times2=75(cos(375)+isin(375))[/tex]

375° is equivalent to 15° (375 - 360 = 15), then the product of z1 and z2 can be finally written as:

[tex]z1\times z2=75(cos(15)+\imaginaryI s\imaginaryI n(15))[/tex]

Then, option A is the correct answer

A true-false test contains 11 questions. In how many different ways can this test be completed. (Assume we don't care about our scores.)Your answer is :

Answers

Let's suppose that 1 = TRUE and 0 = FALSE, we want to find how many combinations we can do with 11 zeros and ones, in fact, it's:

[tex]\begin{gathered} \text{ 000 0000 0000} \\ \text{ 000 0000 0001} \\ \text{ 000 0000 0010} \\ \text{ 000 0000 0011} \\ ... \\ \text{ 111 1111 1111} \end{gathered}[/tex]

To evaluate the number of combinations we can do:

[tex]C=2^{11}[/tex]

2 because we can pick 2 different options (true or false) and 11 because it's the number of questions, then

[tex]\begin{gathered} C=2^{11} \\ \\ C=2048 \end{gathered}[/tex]

We have 2048 different ways that this test can be completed.

Determine if the line passing through A(7,5) and B(-14, -9) is parallel, or perpendicular to the line passing through C(0,1) and D(4, -5).

Answers

To solve this problem, we will use the two pair of points to find the slope of the equation of each line. Then, by comparing these slopes, we can determine either if they are perpendicular or parallel.

Slope calculations

To calcula the slopes, given the pairs of points, we are going to use the following formula: Given points (a,b) and (c,d) the slope of the line that passes through them is given by the formula

[tex]m=\frac{d\text{ - b}}{c\text{ -a}}=\frac{b\text{ - d}}{a\text{ -c}}[/tex]

Let us calculate first the slope of the line that passes through the points (7,5) and (-14,-9). In this case, we have a=7,b=5,c=-14 and d=-9. So we get

[tex]m=\frac{5\text{ - (-9)}}{7\text{ - (-14)}}=\frac{14}{21}=\frac{2\cdot7}{3\cdot7}=\frac{2}{3}[/tex]

Now, let us calculate the slope of the line that passes through the points (0,1) and (4,-5). In this case, we have a=0,b=1,c=4 and d=-5. So we get

[tex]m=\frac{1\text{ -(-5)}}{0\text{ - 4}}=\frac{6}{\text{ -4}}=\text{ -}\frac{3\cdot2}{2\cdot2}=\text{ -}\frac{3}{2}[/tex]

Slope comparison

Now, we compare the slopes to determine if the lines are perpendicular or parallel. Recall that two lines are parallel if they have the same slope and they are perpendicular if the product of their slopes is -1. From our calculations, we can see that the slopes are not equal. Let us confirm that they are perpendicular. To do so, we multiply both slopes. So we get

[tex]\frac{2}{3}\cdot(\text{ -}\frac{3}{2})=\text{ -1}[/tex]

Since their product is -1, this confirms that both lines are perpendicular.

HELP!!
Triangle ABC is shown with exterior ∠z.


triangle ABC with angle A labeled 58 degrees, angle B labeled 44 degrees, and side AC extended with angle z labeled as exterior angle to angle C


Determine m∠z.


136°

102°

78°

58

Answers

The sum of all interior angles of a triangle is 180° thus the measure of the exterior angle m∠K is 102° so option (B) is correct.

What is a triangle?

A triangle is a 3-sided shape that is occasionally referred to as a triangle. There are 3 sides and three angles in every triangle, some of which may be the same.

Triangle is a very common figure to deal with in our daily life.

It is known that the sum of all three angles inside a triangle will be 180°.

So,   m∠A +  m∠B +  m∠C = 180°

m∠C = 180° - 58° -  44°

m∠C = 78°.

The exterior angle of C = 180- 78 = 102°.

Hence "The sum of all interior angles of a triangle is 180° thus the measure of the exterior angle to m∠K is 102°".

For more about triangles,

https://brainly.com/question/2773823

 

#SPJ1

 

Answer:

136

Using the exterior angle theorem,

Step-by-step explanation:

It took 12 men 5 hours to build an airstrip. Working at the same rate, how many additional men could have been hired in order for the job to have taken 1 hour less?

A) Two
B) Three
C) Four
D) Six

Answers

Answer:

B

Step-by-step explanation:

hello the question is if it took 12 men 5 hours to build an age is working at the same rate how many additional men could have been hired in order for the job to have taken 1 hour less ok so we have to find that how many extra may be required to complete the job for a strip 1 hour less than five hours that is 4 hours ok bye have to complete the airstrip in 4 hours and they have to find that how many experiment we have to required for that we will assume that let extra number of number of extra man bhi X show the number of men when we are finishing in it in 4 hours would be 12 + X ok

would be the number of men now and time required would be equal to 1 hour less than 5 hours that is 4 hours ok no from the given data we can say that one cares if strip x 12 men and fibres ok so all vacancy job correct so vacancy job per man are would be 1 divided by 12 in 25 this is the job or the amount of a strip that is completed when

one man works for one hour ok so this is the amount of the job that is done for men power and we have we have this number of men that are not want working and the number of hours that their working for so for one job we will need for one job would be best job per man per hour into number of men into number of hour ok and we have 1 equal to number of Doberman per Rs 1 by 2 11 25 and number of men we have already know that 12 + 6 is the number of men that we will require 12 + X number

forces were less than 5 that is 44 4312 so it would give us 12 + X / 3515 1 to 15 of this site it would give us 12 + X equal to 15 which implies X is equals to 15 - 12 and X is equals to 3 significant required 3 more men to complete the job in Porus dancer is 3 which is given is be in the question make you

I need help with problem 7.Use the figure to find the values of x, y, and z that makes triangle DEF similar to triangle GHF.

Answers

ANSWER

• x = 12

,

• y = 16

,

• z = 7

EXPLANATION

Because the triangles are similar, we have that:

• The ratio between corresponding sides is constant:

[tex]\frac{DE}{GH}=\frac{EF}{GF}=\frac{DF}{HF}[/tex]

• Corresponding angles are congruent:

[tex]\begin{gathered} \angle D\cong\angle H \\ \angle E\cong\angle G \\ \angle F\cong\angle F \end{gathered}[/tex]

We know that the measure of angle E is 16°, so the measure of angle G must be the same because they are congruent,

[tex]16\degree=2(x-4)\degree[/tex]

With this equation, we can find x. First, divide both sides by 2,

[tex]\begin{gathered} \frac{16}{2}=\frac{2(x-4)}{2} \\ \\ 8=x-4 \end{gathered}[/tex]

And then, add 4 to both sides,

[tex]\begin{gathered} 8+4=x-4+4 \\ \\ 12=x \end{gathered}[/tex]

Hence, x = 12.

Now we know that the length of side EF is,

[tex]EF=x-5=12-5=7[/tex]

To find y and z, we will use the proportions we got at the top of this explanation,

[tex]\frac{DE}{GH}=\frac{EF}{GF}=\frac{DF}{HF}[/tex]

Replace with the known values and the expressions with y and z,

[tex]\frac{25}{6z+8}=\frac{7}{14}=\frac{24}{3y}[/tex]

With the first two, we can find z,

[tex]\frac{25}{6z+8}=\frac{7}{14}[/tex]

Simplify the right side,

[tex]\frac{25}{6z+8}=\frac{1}{2}[/tex]

Rise both sides to the exponent -1 - i.e. flip both sides of the equation,

[tex]\frac{6z+8}{25}=2[/tex]

Multiply both sides by 25,

[tex]\begin{gathered} 25\cdot\frac{(6z+8)}{25}=2\cdot25 \\ \\ 6z+8=50 \end{gathered}[/tex]

Subtract 8 from both sides,

[tex]\begin{gathered} 6z+8-8=50-8 \\ 6z=42 \end{gathered}[/tex]

And divide both sides by 6,

[tex]\begin{gathered} \frac{6z}{6}=\frac{42}{6} \\ \\ z=7 \end{gathered}[/tex]

Hence, z = 7.

Finally, with the last two proportions, we can find y,

[tex]\frac{7}{14}=\frac{24}{3y}[/tex]

The first two steps are the same we did to find z: simplify the left side and flip both sides,

[tex]2=\frac{3y}{24}[/tex]

Multiply both sides by 24,

[tex]\begin{gathered} 24\cdot2=24\cdot\frac{3y}{24} \\ \\ 48=3y \end{gathered}[/tex]

And divide both sides by 3,

[tex]\begin{gathered} \frac{48}{3}=\frac{3y}{3} \\ \\ 16=y \end{gathered}[/tex]

Hence, y = 16.

Other Questions
a conductor consists of an infinite number of adjacent wires, each infinitely long and carrying a current i (whose direction is out-ofthe-page), thus forming a conducting plane. How do you know the surface water is not there because the sediment is saturated? a change in state involves a change in the______of the particles An atom of Gold contains 120 neutrons. What is its mass number? What best demonstrates the flow of energy through the environment?(A)Autotroph-Sun-Heterotroph-Decomposer(B)Sun-Autotroph-Heterotroph-Decomposer(C)Sun-Decomposer-Heterotroph-Autotroph(D)Sun-Heterotroph-Autotroph-Decomposer Which diagnostic test measures the level of oxygen and carbon dioxide in the blood?. if the pressure is decreasing at a rate of 50 kg/m2, the temperature is increasing at a rate of 7.2 k/s, and the amount of gas (i.e., the number of moles) remains the same, what is the rate of change of the volume? The distribution of the salaries at a small business is strongly skewed with many people making very modest salaries and a few making very large salaries. Which measure of center, mean or median, is a better indication of a "typical salary at this business?The mean is better because it is not affected by the high salaries.The median is better because it is not affected by the high salaries.It cannot be determined without knowing the actual salaries.Either one would be a good measure of a typical salary because they are the same value.(Plss someone help, this is my last try... ima cry if I fail this sh again) Can someone tell me how to solve it with a picture on how you did it. Teacher told me to show my work. It hard pls Helppp Which expression is equivalent to (1)/(2^(-2)*3^(-2)) ? Do it with reported speech ,Raderman said,The container library project will have a long lasting positive impact on the lives of thousands of young learners There are four types of stem-changing verbs.- TrueO False Pancakes are made with 1 cups water and 1/2What is the ratio of water to milk?cup milk. describe the intertwined labor and civil rights struggles of this period (both black and latinx), the challenges or problems that they confronted in the new deal and wwii eras, and their accomplishments, failures, eventual decline, and legacies left for later activists. Drag each of the crowns into the liquid. Based on what you see, which crown do you think is densest? Explain why you think so. elaborate : how might the seasons affect human activities? the world population was 6.9 billion at the end of 2010 and is predicted to reach 9 billion by the end of 2050.1 (a) assuming the population is growing exponentially, what is the continuous growth rate per year? round your answer to three decimal places. 49 = 7/6u solve for u simplify your answer as much as possible What is the domain of the square root function graphed below?On a coordinate plane, a curve opens down to the right in quadrant 1. The curve starts at (3, 1) and goes through (4, 2) and (7, 3). Any one described what is chemistry of photography your own word?